This would have been a nice 1952's problem

Find the sum of all integral values of 3 3 x 3 2 x + 1 + 3 x + 1 1952 3 \sqrt[3]{3^{3x}-3^{2x+1}+3^{x+1}-1952} where x x is an integer.


The answer is 25.

This section requires Javascript.
You are seeing this because something didn't load right. We suggest you, (a) try refreshing the page, (b) enabling javascript if it is disabled on your browser and, finally, (c) loading the non-javascript version of this page . We're sorry about the hassle.

3 solutions

Michael Tang
May 13, 2014

Suppose that 3 3 x 3 2 x + 1 + 3 x + 1 1952 = n 3 3^{3x} - 3^{2x+1} + 3^{x+1} - 1952 = n^3 for some integer n . n. Then we can write

3 3 x 3 3 2 x + 3 3 x 1952 = n 3 . 3^{3x} - 3 \cdot 3^{2x} + 3 \cdot 3^x - 1952 = n^3.

Now, the critical step is to notice this factorization:

( 3 x 1 ) 3 1951 = n 3 . (3^x-1)^3 - 1951 = n^3.

(Notice the 1 , 3 , 3 1, -3, 3 coefficients on the left-hand side, which was my primary motivation.) Then, rearranging gives ( 3 x 1 ) 3 n 3 = 1951. (3^x-1)^3 - n^3 = 1951. Let a = 3 x 1 , a = 3^x - 1, so we have a 3 n 3 = 1951 a^3 - n^3 = 1951 or

( a n ) ( a 2 + a n + n 2 ) = 1951. (a-n)(a^2+an+n^2) = 1951.

Normally, we would now proceed to try a bunch of factor pairs of 1951 1951 ... but we hope, hope, hope that Daniel was nice to us when writing this problem and won't make us try too many. Indeed, 1951 1951 is prime, the easiest of all! :P

Also, note the following lemma:

Lemma. For all real numbers a , n a, n (whether negative or positive), the following inequality holds: a 2 + a n + n 2 0. a^2+an+n^2 \ge 0.

Proof. If a , n a, n have the same sign (or one/both of them is zero), then a n 0 , an \ge 0, so

a 2 + a n + n 2 a 2 + n 2 0 + 0 = 0. a^2+an+n^2 \ge a^2+n^2 \ge 0+0 = 0.

Otherwise, if a , n a, n have the opposite sign, then a n 0 an \le 0 ; but now we can write

a 2 + a n + n 2 = ( a + n ) 2 a n ( a + n ) 2 0. a^2+an+n^2 = (a+n)^2 - an \ge (a+n)^2 \ge 0.

Thus, in all cases, a 2 + a n + n 2 0. a^2+an+n^2 \ge 0.

So, by the Lemma, there are only two cases to try:

(1) Suppose that a n = 1 a-n=1 and a 2 + a n + n 2 = 1951. a^2+an+n^2 = 1951. We substitute a = n + 1 a = n+1 into the second equation to get ( n + 1 ) 2 + n ( n + 1 ) + n 2 = 1951 , (n+1)^2 + n(n+1) + n^2 = 1951, which simplifies to

3 ( n 2 + n 650 ) = 0. 3(n^2+n-650) = 0.

This has roots n = 25 n = 25 or n = 26. n = -26. But if n = 26 , n = -26, then a = 25 a = -25 and the equation 3 x 1 = 25 3^x - 1 = -25 has no real solutions. Thus, n = 25 , n = 25, so a = 26 , a = 26, so 3 x 1 = 26 x = 3. 3^x-1=26 \Longrightarrow x = 3. The solution x = 3 x = 3 corresponds to the value 25 , 25, since we defined n n to be the value of the expression.

(2) Suppose that a n = 1951 a-n=1951 and a 2 + a n + n 2 = 1. a^2+an+n^2 = 1. We substitute a = n + 1 a = n+1 into the second equation to get ( n + 1951 ) 2 + n ( n + 1951 ) + n 2 = 1 , (n+1951)^2 + n(n+1951) + n^2 = 1, which simplifies to

3 n 2 + 5853 n + 3806400 = 0. 3 n^2+5853 n+3806400 = 0.

This is a mess, and has no real roots for n , n, so there are no solutions in this case.

So, the only value of x x is x = 3 , x=3, which corresponds to the value 25 . \boxed{25}. \square

That was INCREDIBLE! Loved this solution and really improved my math skills xD

Felipe Magalhães - 7 years ago

This solution is perfect. Double Thumbs up.

See, I think waiting for someone else to post a solution before I do is the better strategy. Me posting a solution immediately deters others from doing so.

Daniel Liu - 7 years, 1 month ago

Solved the same way!!Nice!

Eddie The Head - 7 years, 1 month ago

Very nice!

Must say, an extremely astute question and an equally wonderful solution!!!!!!!

Shreyansh Vats - 7 years ago

Here's a semi-open question: Why was it the number 1952 1952 at the beginning? What, if anything, was special about 1952 1952 here? What conditions did that number, say k , k, need to satisfy in order for there to be an integer solution using the method above?

Michael Tang - 7 years, 1 month ago

Log in to reply

It's one more than a prime number, I guess. But great solution! :D

Finn Hulse - 7 years, 1 month ago

Log in to reply

Thanks :) (Also, the part at the end, with the quadratic, should work out to an integer root.)

Michael Tang - 7 years, 1 month ago

Another good solution would be a bounding argument. for big enough n n , we must have that n 3 ( n 1 ) 3 > 1951 n^3-(n-1)^3 > 1951 , where n n in this case is 3 x 1 3^x-1 . We find that if x > 3 x > 3 , then n 3 ( n 1 ) 3 > 1951 n^3-(n-1)^3 > 1951 so the expression can never be a perfect cube. Thus we just need to check the values of x = 0 , 1 , 2 , 3 x=0,1,2,3 .

Daniel Liu - 7 years ago

in case 2 it should be a=n+1951 and not a=n+1.

yash gupta - 7 years, 1 month ago

Log in to reply

Oops, sorry. That happens when you copy-paste between cases :P

Michael Tang - 7 years, 1 month ago

i did same

Dev Sharma - 5 years, 5 months ago

Tanks Dood for the Awesome sol :)

HariShankar PV - 7 years ago
Bogdan Simeonov
May 14, 2014

For x>3 we have ( 3 x 1 ) 3 ( 3 x 2 ) 3 > 1951 (3^x-1)^3-(3^x-2)^3>1951 .That leaves x=3,2,1.Done :D

Could hardly fpllow.Plz xpln!

Chandrachur Banerjee - 7 years, 1 month ago

Log in to reply

The thing under the radical is equal to ( 3 x 1 ) 3 1951 (3^x-1)^3-1951 .We want this to be an integer, call it a.Then ( 3 x 1 ) 3 a 3 = 1951 (3^x-1)^3-a^3=1951

But for x > 3 x>3 we would have

( 3 x 1 ) 3 ( 3 x 2 ) 3 > 1951 (3^x-1)^3-(3^x-2)^3>1951 , so we would obviously have no solutions.

Now we are left with checking for 1,2 and 3.

Bogdan Simeonov - 7 years ago
Finn Hulse
May 13, 2014

I think the reason this problem would have been tough is if you were using a crappy calculator. Because I noticed for almost all x x 's, the formula produced a number that was like, insanely close to a whole number, like at x = 12 x=12 , the result was 531439.99999999769735209398753712937322597604322742013 531439.99999999769735209398753712937322597604322742013 . But I just kept plugging in values. The only real "trick" I used was realizing that x x had to be greater than 2 2 because otherwise the part under the radical would be negative. Plugging in x = 3 x=3 produces 25 \boxed{25} which I guess is the only truly integral solution. I'll be waiting for a good solution though, @Daniel Liu . Nice problem! :D

It's super hot down here at SD...so I don't feel like writing a full solution.

Here are some hints:

  1. Notice the ( a 1 ) 3 (a-1)^3 factorization.

  2. 1951 is prime

Xuming Liang - 7 years, 1 month ago

Log in to reply

Mmm I see what you're saying. Interesting.

Finn Hulse - 7 years, 1 month ago

It's fine if the part under the radical (the radicand) is negative, since we're taking a cube root...

Michael Tang - 7 years, 1 month ago

Log in to reply

Oh yeah. Darn. Whatever though.

Finn Hulse - 7 years, 1 month ago

Your calculator is giving you a number a tiny bit smaller than 3 x 1 3^x - 1 for large enough values of x x . E.g. 3 12 1 = 531440 3^{12} - 1= 531440 . This is because the expression simplifies to ( 3 x 1 ) 3 1951 3 \sqrt[3]{(3^x-1)^3-1951} , and for large enough values of x x , subtracting 1951 1951 doesn't change the cube root very much.

This gives some insight into the actual solution of the problem.

Patrick Corn - 7 years, 1 month ago

Daniel? I'm waiting for a freaking boss solution... :D

Finn Hulse - 7 years, 1 month ago

Log in to reply

From now on, I'm waiting for someone else to maybe post a solution before I do. Give others a chance.

Daniel Liu - 7 years, 1 month ago

Log in to reply

Aw nartz. But you do have one, right? Is it really cool?

Finn Hulse - 7 years, 1 month ago

Log in to reply

@Finn Hulse Basically the same as Michael's.

Daniel Liu - 7 years, 1 month ago

Log in to reply

@Daniel Liu Oh epic. How did you think this up? Did you just choose a number 1 greater than a prime?

Finn Hulse - 7 years, 1 month ago

Log in to reply

@Finn Hulse No, I had to also choose the number such that it actually gives an integer solution. Not all numbers that are one more than a prime work.

Daniel Liu - 7 years ago

Log in to reply

@Daniel Liu 'Tis a problem. But still, excellent problem. BTW what did you get on USAJMO? Did you make USAMO? :O

Finn Hulse - 7 years ago

Log in to reply

@Finn Hulse I didn't make USAMO because I took the USAJMO. And I got a 9 on the JMO.

Daniel Liu - 7 years ago

Log in to reply

@Daniel Liu Wait 9 out of what?

Finn Hulse - 7 years ago

Log in to reply

@Finn Hulse 9 out of 42.

Daniel Liu - 7 years ago

Log in to reply

@Daniel Liu Ooh that stings. @Akshaj Kadaveru got a 21/42. :O

Finn Hulse - 7 years ago

Done with my solution :)

Michael Tang - 7 years, 1 month ago

Log in to reply

@Michael Tang EPIC! This solution was so amazing. :D

Finn Hulse - 7 years, 1 month ago

I did it the same way, and then found that x = 3 works, and then I figured the problem couldn't go on forever, so I stopped at 25.

Asher Joy - 7 years, 1 month ago

Log in to reply

Yah. This problem was supposed to be Level 5. @Daniel Liu what are the stats as of right now for who has solved it?

Finn Hulse - 7 years, 1 month ago

Log in to reply

29 views, 6 attempts, 6 solvers. It's too easy to guess the right solution, I guess.

Daniel Liu - 7 years, 1 month ago

Log in to reply

@Daniel Liu Yea this is a problem. @Calvin Lin how many of the 7 solvers currently, do you suppose, actually solved the question instead of trying the first few x x and then guessing that there aren't any more solutions? Is there any way to prevent guessing?

Daniel Liu - 7 years, 1 month ago

Log in to reply

@Daniel Liu It is a shortcoming of asking for numeral answers (which are easy to evaluate in scale) as opposed to proofs (which are hard to individually judge).

There is no easy way to prevent guessing, or to prevent "I threw it into Python".

Calvin Lin Staff - 7 years, 1 month ago

@Daniel Liu @Daniel Liu I did this the same way as @Michael Tang . Is it still 19 19 attempts and 19 19 solvers?

Trevor B. - 7 years, 1 month ago

it's really interesting............. :-p

ÇĦêxøstylø Durga - 7 years, 1 month ago

0 pending reports

×

Problem Loading...

Note Loading...

Set Loading...